K
Khách

Hãy nhập câu hỏi của bạn vào đây, nếu là tài khoản VIP, bạn sẽ được ưu tiên trả lời.

26 tháng 2 2018

Câu hỏi của Hoa Thân - Toán lớp 8 - Học toán với OnlineMath

Em tham khảo bài tương tự tại đây nhé.

18 tháng 10 2016

Có sai đề không bạn

18 tháng 10 2016

Ta có :   D=(x^2 -  2x  +  1)  +  (4x^2 +  4x +1)

                 = x^2 - 2x + 1 + 4x^2 +4x +  1

                 = 5x^2 + 2x + 2

                 =5(x^2  +  2/5x + 2/5)

                 =5(x^2 + 2/5x + 1/25 + 9/25)

                 =5(x^2 +2/5x +1/25) + 9/5 >= 9/5

 Vậy MinD=9/5 khi x=-1/5

18 tháng 10 2016

Mình bổ sung 1 chút ở chỗ 5(x^2 + 2/5x + 1/25) + 9/5 = 5(x+1/5)^2  + 9/5 >= 9/5

18 tháng 10 2016

<=> [ ( x + 2) ( x + 5) ] [ ( x + 3) ( x + 4 ) ] - 24 = ( x+ 7 x + 10 ) ( x2 + 7 + 12 ) - 24 (1)  

Đặt x2 +  7x + 11 = t

=> (1) <=> ( t - 1 ) ( t + 1 ) - 24 = t2 -1 - 24 = t2 - 25 = ( t - 5 ) ( t + 5)

<=> ( x2 + 7x + 11 - 5 ) ( x+ 7x + 11 + 5 ) = ( x2 + 7x + 6 ) ( x2 + 7x + 16 ) 

= ( x + 1 ) ( x + 6 ) ( x2 + 7x + 16 )

18 tháng 10 2016

= (x^2 +7x + 10)(x^2 +7x + 12) -24

Đặt x^2 + 7x + 10 = t

=> t(t+2)-24 = t^2 +2t -24 = (t+4)(t-6)

Trả lại biến cũ , ta được :

(x^2 + 7x + 14)(x^2 + 7x + 4)

20 tháng 10 2016

ngu nguoi bai nay ma ko biet giai

18 tháng 10 2016

Trước hết bạn chứng minh :  \(a^2+b^2+c^2\ge\frac{\left(a+b+c\right)^2}{3}\Rightarrow a+b+c\le\sqrt{3\left(a^2+b^2+c^2\right)}\) (Chứng minh bằng biến đổi tương đương)

Áp dụng BĐT AM-GM ta có : \(\frac{1}{2-a}+\frac{1}{2-b}+\frac{1}{2-c}\ge\frac{9}{6-\left(a+b+c\right)}\ge\frac{9}{6-\sqrt{3\left(a^2+b^2+c^2\right)}}=\frac{9}{6-3}=3\)

18 tháng 10 2016

Dễ thấy \(0< a,b,c< 2\)

Ta có:

\(\frac{1}{2-a}\ge\frac{a^2+1}{2}\Leftrightarrow a\left(a-1\right)^2\ge0\)

Tương tự với các cái tương tự, ta được:

\(\frac{1}{2-a}+\frac{1}{2-b}+\frac{1}{2-c}\ge\frac{a^2+1+b^2+1+c^2+1}{2}=3\)(Đpcm)

Dấu = khi a=b=c=1

18 tháng 10 2016

x = 2 đó bạn